Retirement Planning Sample Exam

अब Quizwiz के साथ अपने होमवर्क और परीक्षाओं को एस करें!

Which of the following is an example of a qualified retirement plan? A) Rabbi trust B) 401(k) plan C) Nonqualified stock option plan D) ESPP

A 401(k) plan is a qualified plan. All of the others are not qualified retirement plans.

Charles, a single 29-year-old, deferred 2% of his salary, or $2,000, into a 401(k) plan sponsored by his employer during 2020. What is the maximum deductible IRA contribution Charles can make during 2020? A) $0 B) $1,000 C) $4,000 D) $6,000

A) $0 Charles cannot make a deductible IRA contribution for the year because he is an active participant in a qualified plan with an AGI of at least $100,000 ($2,000 / 2%), which exceeds the phaseout limits for 2020.

Natalie is a secretary at JKL Law Firm. JKL provides her with free sodas at her discretion. Natalie estimates that she drinks $20 worth of sodas per month. How much must Natalie include in her annual gross income related to the sodas? A) $0 B) $20 C) $200 D) $240

A) $0 The value of employer-provided de minimis fringe benefits may be excluded from an employee's gross income. After 2017, however, the employer's deduction for the sodas is limited to 50 percent due to TCJA 2017.

Vance has a vested account balance in his employer-sponsored qualified profit-sharing plan of $40,000. He has two years of service with his employer. The plan follows the least generous graduated vesting schedule permitted for a profit-sharing plan under the Pension Protection Act of 2006. If Vance has an outstanding loan balance of $15,000 within the prior 12 months, what is the maximum loan amount Vance could take from this qualified plan, assuming the plan permitted loans? A) $5,000 B) $20,000 C) $40,000 D) $50,000

A) $5,000 The maximum loan an individual can take is the lesser of $50,000 or 50% of their vested account balance. In this case, Vance has a vested account balance of 20% (2 years of service with a 2-to-6-year vesting schedule) of $200,000, or $40,000; so the maximum loan would be 50% of $40,000, or $20,000. Since Vance had an outstanding loan balance of $15,000 within 12 months, the maximum loan available must be reduced by $15,000. In this case, the maximum loan Vance could take from the qualified plan is $5,000 ($20,000 − $15,000).

Cheque Company has 100 eligible employees and sponsors a defined-benefit pension plan. The company is unsure if it is meeting all of the testing requirements. How many employees (the minimum) must be covered by Cheque Company's defined-benefit pension plan for the plan to conform with ERISA? A) 40 B) 50 C) 70 D) 100

A) 40 The 50/40 rule requires that defined-benefit plans cover the lesser of 50 employees or 40% of all eligible employees. In this example, 40% of 100, or 40 employees, would be the lesser of these two amounts.

All of the following statements concerning IRA planning considerations are correct EXCEPT: A) A Roth IRA contribution is likely to result in a greater long-term accumulation than a deductible IRA contribution if the participant's marginal tax bracket is higher at the time of the contribution than the time of the distribution. B) A Roth IRA typically has better tax benefits than a nondeductible contribution to a traditional IRA. C) An employee contribution to a 401(k) plan may result in a better long-term accumulation than a Roth IRA contribution if the employer matches the 401(k) salary deferral. D) A Roth IRA conversion can be a good estate-planning tool for those who will be subject to estate or inheritance taxes.

A) A Roth IRA contribution is likely to result in a greater long-term accumulation than a deductible IRA contribution if the participant's marginal tax bracket is higher at the time of the contribution than the time of the distribution. The opposite is true. The Roth IRA results in a greater accumulation than a traditional IRA if the tax rate is lower at the time of the contribution than the time of the distribution.

All of the following statements about SEP and SIMPLE IRAs are true EXCEPT: A) A SEP requires the plan sponsor to provide at least a 100% match up to 3% of all employee deferrals. B) A SEP plan can be established by employers who employ more than 100 employees who earn $5,000 or more during the preceding calendar year. C) SIMPLEs can be either contributory or noncontributory plans, whereas SEP plans are always noncontributory. D) An employer who wants to share the responsibility of retirement plan funding should establish a SIMPLE rather than a SEP.

A) A SEP requires the plan sponsor to provide at least a 100% match up to 3% of all employee deferrals. Statement (A) is incorrect because a SEP is a noncontributory plan that does not receive employee deferral contributions. Statement (B) is correct as the 100 employee limit applies to SIMPLEs. Statement (C) is correct as SIMPLEs can be either contributory or noncontributory and SEPs are noncontributory. Statement (D) is correct because a SIMPLE can be a contributory plan, thus receiving allocations from employee deferrals, and sharing the burden of plan funding with the employee.

Of the following statements regarding target-benefit pension plans, which is true? A) A target-benefit pension plan is a money-purchase pension plan with a funding formula that actuarially considers age and salary. B) The plan sponsor of a target-benefit pension plan guarantees that the participant will receive an amount that is expected to be the "target benefit" amount, at his retirement. C) Plan participants of a target-benefit pension plan can choose to invest up to 20% of their account assets into employer stock. D) The plan sponsor guarantees an earnings rate of at least 6 percent for the contributions made to target-benefit pension plans.

A) A target-benefit pension plan is a money-purchase pension plan with a funding formula that actuarially considers age and salary. Statement (A) is true. Statements (B), (C), and (D) are false. The plan sponsor is required to fund the participant of a target-benefit pension plan's separate account each year with the actuarially equivalent present value of the "target." The "target" is the expected benefit that the plan sponsor forecasts (at inception) that the participant will receive at retirement, but the sponsor does not make any guarantees on the benefit payable from the plan. Statement (C) is incorrect because no qualified plan may invest more than 10% of its assets into employer securities. Statement (D) is incorrect because the plan sponsor does not guarantee a minimum investment return on participant account assets.

Which of the following statements concerning HSAs is correct? A) HSA distributions that are used to pay for non-medical expenses are subject to a 20% penalty. B) Any HSA distributions not used for medical expenses are subject to long-term capital gains taxes. C) If HSA contributions are less than the prescribed limits, the account earnings are included in the account holder's gross income for federal income tax purposes. D) A married couple may contribute to up to one medical FSA and up to one HSA in any given year.

A) HSA distributions that are used to pay for non-medical expenses are subject to a 20% penalty. HSA distributions used to pay for non-medical expenses are subject to a 20% penalty as well as federal income tax (not, as statement B states, long-term capital gains taxes). Statement (C) is incorrect because, under the circumstances described, the account earnings would not be included in the account holder's gross income for federal income tax purposes. Statement (D) is incorrect because if one spouse is covered by an FSA, this is treated as both spouses are being covered. If eligible, a married couple must select between an FSA and an HSA. An exception is if the FSA is "limited purpose" (covers dental and vision only).

Generally, when must employees include the value of a fringe benefit in their gross income? I. When they were given the choice to decline the fringe benefit II. When the benefit is provided for the convenience of the employer A) I only B) II only C) Both I and II D) Neither I nor II

A) I only Generally, employees must include the value of a fringe benefit in their gross income when they are given the choice to decline the benefit but took it anyway. For example, an employee who is given the option (but not the obligation) to live in employer-provided lodging would likely be required to include the value of that lodging in his or her gross income. Statement II is incorrect because benefits provided for the convenience of the employer generally are not included in the employee's gross income.

Which of the following is true regarding negative elections? I. A negative election is a provision whereby the employee is deemed to have elected a specific deferral unless the employee specifically elects out of such election in writing. II. Negative elections are no longer approved by the IRS.III. When an employer includes a negative election in its qualified plan, the employer must also provide 100% immediate vesting. A) I only B) I and III C) II and III D) I, II, and III

A) I only Negative elections are approved by the IRS, and they are available for both current and new employees. Negative elections do not require 100% immediate vesting.

Wanka Factory has 100 non-excludable employees, 10 of whom are highly compensated. Eight of the 10 highly compensated and 63 of the 90 non-highly compensated employees are covered under Wanka's qualified plan. The average accrued benefits for the highly compensated are 4% and the average accrued benefit for the non-highly compensated is 1.5%. Which of the following statements is true regarding coverage? I. The plan passes the ratio percentage test. II. The plan passes the average benefits percentage test. A) I only B) II only C) Both I and II D) Neither I nor II

A) I only The ratio percentage test compares the percentage of covered non-highly compensated to the percentage of covered highly compensated employees. The ratio must be greater than or equal to 70% for the plan to pass the ratio percentage test. The calculation for Wanka's qualified plan is as follows: NHC = 63/90 = 70% HC = 8/10 = 80% 70%/80% = 87.5% Wanka's plan passes the ratio-percentage test requirement of 70%. The average benefits percentage test requires the average benefit of the non-highly compensated employees to be at least 70% of the average benefit of the highly compensated. Wanka's plan does not satisfy the average benefits test because the average benefit of the non-highly compensated compared to the average benefit of the highly compensated is less than 70% (1.5%/4% = 37.5% = fail).

Which of the following is (are) elements of an effective waiver for a preretirement survivor annuity? I. The waiver must be signed within six months of death. II. The waiver must be signed by a plan participant. III. The waiver must be notarized or signed by a plan official. A) III only B) I and II C) II and III D) I, II, and III

A) III only Statement III is correct. Statement II is incorrect, as only the nonparticipant spouse must sign the waiver. Statement I is unfounded.

All of the following statements concerning a qualified plan's summary plan description (SPD) are correct EXCEPT: A) It typically is used to persuade employees to join the plan. B) It is an easy-to-read booklet that explains the plan to the participants. C) It may be prepared by the financial services professional, the insurer, or the employer. D) It must be issued to participants within 120 days after the plan is adopted by the board of directors.

A) It typically is used to persuade employees to join the plan. The SPD cannot be used as a sales piece. The other statements are correct

All of the following statements concerning the Social Security system are correct EXCEPT: A) The Social Security retirement benefit is payable at normal retirement age, with reduced benefits available as early as age 59 ½, to anyone who has obtained at least a minimum amount of Social Security benefits. B) Disability benefit recipients must have a severe physical or mental impairment that is expected to either prevent them from performing a "substantial" amount of work for at least a year or result in death. C) The family benefit is provided to certain family members of workers eligible for retirement or disability benefits. D) Survivors benefits apply to those family members listed for family benefits, and these may also include the worker's parents if the worker was their primary means of support.

A) The Social Security retirement benefit is payable at normal retirement age, with reduced benefits available as early as age 59 ½, to anyone who has obtained at least a minimum amount of Social Security benefits. Age 62 is the minimum retirement age except for widows and widowers who are eligible for reduced benefits at age 60.

Which of the following statements concerning a Sec. 423 employee stock-purchase plan is correct? A) The effective discount can be substantially higher than 15 percent off of the stock's price. B) The plan can be limited to a few highly compensated executives. C) Participants pay taxes at the time the stock is purchased. D) Like nonqualified stock options, the entire value of the discount is taxed as ordinary income.

A) The effective discount can be substantially higher than 15 percent off of the stock's price. If the price of the stock rises over the purchase period and the price is 15 percent off the market price at the beginning of the period, the discount can vastly exceed 15 percent of the market price of the stock at the time of the purchase. Statement (B) is incorrect because these plans must cover most full-time employees. Statement (C) is incorrect because taxes are not paid until the stock is sold. Statement (D) is incorrect because only a portion of the value is taxed as ordinary income.

Which of the following statements regarding the plan sponsor of a money-purchase pension plan is correct? A) The plan sponsor is required to make an annual contribution to the plan. B) The excess earnings of a money-purchase pension plan are returned to the plan sponsor. C) The plan sponsor generally bears the investment risk of the plan assets. D) A plan sponsor with fluctuating cash flows would adopt a money-purchase pension plan.

A) The plan sponsor is required to make an annual contribution to the plan. A money-purchase pension plan is a defined-contribution pension plan; therefore, it requires an annual contribution by the plan sponsor. Statements (B) and (C) are incorrect because the participant in a money-purchase pension plan generally bears all of the investment risk and benefit of the plan assets. Statement (D) is incorrect because an employer with fluctuating cash flows would not choose a money-purchase pension plan because of the mandatory funding requirement.

Roger and Robin were happily married until Roger fell in love with Sam. As a result, Roger and Robin have agreed they need to get a divorce. As part of the process, the court has provided a domestic relations order that calls for Robin's profit-sharing plan to be divided into equal portions such that Roger will have his own account with half of the value of the retirement account. What type of approach has been taken? A) The separate interest approach B) The split payment approach C) The shared payment approach D) The divided account approach

A) The separate interest approach The separate-interest approach calls for splitting a retirement account into two separate accounts. Each party is free to act with regard to his or her separate account without the interference or consent of the other party. There is no such term as a split-payment approach or divided account approach.

Kevin established a Roth IRA and named his wife, Jennifer, as the sole beneficiary on the beneficiary designation form. Ten years later, he and Jennifer divorced. Kevin updated his will to leave all of his assets to his son, Brian. How will the IRA be distributed at Kevin's death? A) To Jennifer B) To Brian C) To Brian and Jennifer equally D) To Kevin's estate - the executor will have to sort out the situation

A) To Jennifer The beneficiary form controls the distribution of IRA assets and not the will.

Joe Bob receives stock options (ISOs) with an exercise price of $18 when the stock is trading at $18. Joe Bob exercises these options two years after the date of the grant when the stock price is $39 per share. Which of the following statements is correct? A) Upon exercise, Joe Bob will have no regular income for tax purposes. B) Joe Bob will have W-2 income of $21 per share upon exercise. C) Joe Bob will have $18 of AMT income upon exercise. D) Joe Bob's adjusted basis for regular income tax will be $39 at exercise.

A) Upon exercise, Joe Bob will have no regular income for tax purposes. Joe Bob does not have income at the date of exercise. Joe Bob's adjusted basis will be $18. The AMT income, if applicable, is equal to the difference between the fair market value and the exercise price ($39 − $18 = $21).

Steve is self-employed as a marketing consultant. He works primarily with start-up internet companies helping to develop corporate brand programs. Several years ago, he established a 401(k) profit-sharing plan and has accumulated $385,000 in the plan. Which of the following forms should he file to meet his compliance requirements? A)Form 5500 EZ B)Form 5500 SF C)Form 5500 D)He does not need to file a Form 5500 of any type.

A)Form 5500 EZ He must file Form 5500 EZ since it is a one-participant plan and total assets exceed $250,000. If assets were below this threshold, he would not have to file the form.

Christian wants to retire in 15 years when he turns 65. Christian wants to have enough money to replace 75% of his current income less what he expects to receive from Social Security at the beginning of each year. He expects to receive $18,000 per year from Social Security in today's dollars. Christian is aggressive and wants to assume an 8% annual investment rate of return and that inflation will be 3% per year. Based on his family history, Christian expects that he will reach the age of 95. If Christian currently earns $80,000 per year and he expects his raises to equal the inflation rate, how much does he need at retirement to fulfill his retirement goals? A) $1,022,807 B) $1,072,458 C) $1,559,131 D) $1,583,152

B) $1,072,458 Step 1: Estimate retirement needs in today's dollars ($80,000 x 0.75) - $18,000 = $60,000 - $18,000 = $42,000 Step 2: Inflate the need to the beginning of retirement PV: <42,000> I/YR: 3 N: 15 PMT: 0 FV: 65,435 Step 3: Determine the funding need at retirement age BEGIN MODE PMT: 65,435 I/YR: [(1.08/1.03) - 1] x 100 = 4.8544 N: 30 FV: 0 PV: <1,072,458>

Carolyn is planning for her retirement. She is currently 37 years old, plans to retire at age 62, and hopes to live until age 97. She currently earns $100,000 per year and anticipates needing 80% of her income during retirement. Carolyn would like to have the same purchasing power in retirement savings at age 97 as she does at age 62 when she retires. She anticipates that Social Security will provide her with $15,000 per year starting at age 62, leaving her with required savings to provide $65,000 annually during retirement ($100,000 × 0.80 − $15,000). She believes she can earn 11% on her investments, and that inflation will be 2% per year. How much must Carolyn save at the end of each year if she wants to make her last savings payment at age 62 to meet her retirement goal, assuming she does not plan to leave a legacy or bequest? A) $4,759 B) $10,899 C) $11,464 D) $11,566

B) $10,899 Step 1: Estimate retirement needs in today's dollars $65,000 Step 2: Inflate the need to the beginning of retirement PV: <65,000> I/YR: 2 N: 25 PMT: 0 FV: 106,639 Step 3b: Determine the funding need at retirement age BEGIN MODE PMT: 106,639 I/YR: [(1.11/1.02) - 1] x 100 = 8.8235 N: 35 FV: 0 PV: <1,247,036> Step 4: Determine the required annual savings amount END MODE FV: 1,247,046 N: 25 I/YR: 11 PV: 0 PMT: <10,899>

Benjamin retired at age 62 and began to collect Social Security retirement benefits. After a year of playing 3 rounds of golf per day, Ben is bored and decides to go back to work part-time. In 2020 (when Benjamin is aged 63), he earns $28,240 at his part-time job. His Social Security benefit before any adjustments for 2020 is $18,000. What will his actual Social Security benefit be for 2020? A) $10,000 B) $13,000 C) $14,667 D) $18,000

B) $13,000 The earnings reduction applies, and Ben will lose $1 in Social Security benefits for every $2 over $18,240 (in 2020). Since his earnings are $10,000 over the limit, Ben will lose $5,000 of Social Security Benefits, reducing his total benefit to $13,000.

Milton, age 38, earns $170,000 per year. His employer, Dumaine Consulting, sponsors a qualified profit-sharing 401(k) plan and allocates all plan forfeitures to the remaining participants. If in the current year, Dumaine Consulting makes a 20% contribution to all employees and allocates $4,000 of forfeitures to Milton's profit-sharing plan account, what is the maximum Milton can defer to the 401(k) plan in 2020? A) $0 B) $19,000 C) $20,000 D) $25,000

B) $19,000 The maximum annual addition to qualified plan accounts on behalf of Milton is $57,000 for the plan year. The maximum limit on annual additions is comprised of employer contributions, plan forfeiture allocations, and employee deferrals. If Dumaine contributes $34,000 ($170,000 × 20%) to the profit-sharing plan account and Milton receives $4,000 of forfeitures, he may only defer $19,000 ($57,000 − $34,000 − $4,000) before reaching the $57,000 limit.

Perry operates In-N-Out Pharmacy, a sole proprietorship. In-N-Out sponsors a profit-sharing plan. Perry had net income of $205,000 and paid self-employment taxes of $30,000 (assumed) during the year. If Perry makes a contribution on behalf of all of his employees of 15% of salary to the profit-sharing plan, how much is the contribution to the profit-sharing plan on behalf of Perry? A) $22,820 B) $24,776 C) $28,500 D) $30,750

B) $24,776 $205,000 Net Income ($15,000) Less 1/2 SE Tax $190,000 Net SE Income × 0.1304 (0.15/1.15) $24,776 Contribution

Susan's new employer is expecting to contribute $5,000 annually toward her pension account. She also has an option to include a whole-life insurance policy within that account. Given her current age, the level annual premiums would run $10 per $1,000 of death benefit. What is the maximum policy death benefit that her employer can purchase for Susan? A) $125,000 B) $250,000 C) $375,000 D) $500,000

B) $250,000 Susan's pension account starts out empty but will gain $5,000 annually of employer contributions. According to the 25%/50% test, the aggregate of whole-life premiums can't exceed 50% of employer contributions, or $2,500 annually. Since this level premium policy costs $10 annually per $1,000 of death benefit, the maximum benefit is ($2,500 / $10) × $1,000 = $250,000. The 100 to 1 test is not applicable here because no defined benefit was indicated in the question.

Bruce is single, retired, and received Social Security retirement benefits of $18,000 this year. His other income consists of $15,000 from his former employer's pension plan, $10,000 in tax-exempt municipal bond interest, and $1,000 in dividends. How much of Bruce's Social Security Benefit will be subject to income tax? A) $0 B) $5,350 C) $9,000 D) $15,300

B) $5,350 Bruce's MAGI = $15,000 (pension) + $10,000 (tax-exempt interest) + $1,000 (dividends) = $26,000. One-half of his Social Security benefit is $9,000. Since Bruce's MAGI + ½ of the Social Security benefits received is $35,000, or $1,000 over the second hurdle for Social Security taxation of a single person, the amount subject to tax would be the lesser of: (1) 85% × $18,000 (Social Security Benefit) = $15,300(2) 85% × [$26,000 + $9,000 - $34,000] = $850 plus the lesser of $4,500, or (2a) 50% x $18,000 = $9,000, or (2b) 50% x [$26,000 + $9,000 - $25,000] = $5,000(3) Or a total of $850 + $4,500 = $5,350. The lesser of $15,300 or $5,350.

Jim, aged 51, made a $100,000 direct rollover from the after-tax portion of his workplace profit-sharing account into a Roth IRA. He had an adjusted basis in the after-tax account of $150,000. The fair market value of the account at the time of the rollover was $500,000. Calculate the taxable amount of the Roth conversion and any applicable penalty. A) $100,000 taxable, $10,000 tax penalty B) $70,000 taxable, $0 tax penalty C) $70,000 taxable, $7,000 tax penalty D) $30,000 taxable, $0 tax penalty

B) $70,000 taxable, $0 tax penalty Direct rollovers are exempt from the section 72(t) early withdrawal penalty tax, so the penalty must be $0. To calculate the amount of the Roth conversion contribution that is tax-free, the adjusted basis in the plan is divided by the fair market value of the plan as of the day of the distribution. This ratio is then multiplied by the gross rollover amount. Thus $30,000 of the $100,000 distribution is the return of the adjusted taxable basis: ($150,000 / $500,000) × $100,000. Accordingly, $70,000 ($100,000 − $30,000) will be subject to income tax.

Which of the following vesting schedules may a top-heavy, qualified cash-balance plan use? A) 1- to 4-year, graduated B) 35% after 1 year, 70% after 2 years, and 100% after 3 years C) 2- to 6-year, graduated D) 4-year, cliff

B) 35% after 1 year, 70% after 2 years, and 100% after 3 years As a result of the Pension Protection Act of 2006, cash-balance plans must vest at least as fast as a three-year cliff-vesting schedule. This is different than the rule that applies to traditional defined benefit plans. The only choice that is possible is option (B).

Individual accounts in 403(b) plans may not be established in which one of the following forms? A) A contract provided through an insurance company called an annuity contract. B) An account invested in individual stocks in companies in the S&P 500. C) An account for church employees that is a retirement income account, which invests in either annuities or mutual funds. D) An account invested in mutual funds only, referred to as a custodial account.

B) An account invested in individual stocks in companies in the S&P 500. Individual accounts in 403(b) plans must be established as noted in Statements (A), (C), or (D), and thus investing in individual stocks in S&P 500 companies is not allowed. Such stocks may be owned through mutual funds only.

Which of the followings statements regarding 403(b) plans is true? A) Assets within a 403(b) plan may be invested in individual stocks. B) Assets within a 403(b) plan may be invested in mutual funds. C) All 403(b) plans must pass the ADP test. D) In certain situations, a participant of a 403(b) plan can defer an additional $25,000 (catch-up) to a 403(b) plan in a single plan year.

B) Assets within a 403(b) plan may be invested in mutual funds. Only a contributory 403(b) plan has to pass the ADP test. 403(b) plans are not required to meet the ADP test, but 403(b) plans with employer contributions are required to meet the ACP test. The catch-up amount is $15,000 not $25,000.

Which of the employee fringe benefits listed below, if provided by the employer, are both deductible by the employer and not included in an employee's gross income after 2017? I. Business periodical subscriptions II. Season tickets to professional football games III. Parking provided near its business (employer pays $90 per month) IV. The use of an on-premises athletic facility (value of $180 per employee per month) A) II only B) I and IV C) I, III, and IV D) I, II, III, and IV

B) I and IV Season tickets to professional football games are includible in the gross income of the employee receiving the tickets. Periodicals and athletic facilities are both deductible and not included in gross income. Effective with TCJA 2017, season tickets to professional football games are deductible by the employer as compensation expense as long as the cost is includible in the gross income of the employee receiving the tickets. Qualified transportation fringe benefits are not deductible by the employer, as a result of the 2017 Tax Cuts and Jobs Act.

Which of the following statements concerning long-term care is (are) correct? I. Social Security provides many benefits, including long-term care, although it is limited to the participant and the participant's spouse. II. Medicare is inadequate to cover long-term care, in that it will not cover custodial care, if that is all that is needed. A) I only B) II only C) Both I and II D) Neither I nor II

B) II only Statement I is false. Social Security does not provide for long-term care. Statement II is true.

Mike was awarded 1,000 shares of restricted stock in B Corp at a time when the stock price was $14. Assume Mike properly makes an 83(b) election at the date of the award. The stock vests 2 years later at a price of $12 and Mike sells it then. What are Mike's tax consequences in the year of sale? A) Mike has W-2 income of $12,000. B) Mike has a long-term capital loss of $2,000. C) Mike has W-2 income of $14,000. D) Mike has a $12,000 long-term capital gain.

B) Mike has a long-term capital loss of $2,000. In the year of sale, Mike will have a long-term capital loss of $2,000 ($14,000 − $12,000) because his right to the stock was vested. When 83(b) is elected, losses are permitted after the right to the stock has vested.

All of the following are permitted rollovers EXCEPT: A) 401(k) to Roth 401(k) B) Roth 401(k) to Traditional IRA C) 403(b) plan to a qualified plan D) Qualified plan to a governmental 457(b) plan

B) Roth 401(k) to Traditional IRA Roth 401(k) plans can be rolled over only to another similar plan or to a Roth IRA; rollovers into traditional IRAs are not permitted. All of the other rollovers are permitted.

Cindy Sue has been with CS Designs, Inc., for five years. CS Designs has a deferred compensation plan to provide benefits to key executives only. CS Designs contributed $400,000 into a trust for Cindy Sue's benefit under the company's deferred compensation plan. The plan requires that executives must work for the company for 10 years before any benefits can be obtained from the plan. Cindy Sue has come to you to determine when she will be subject to income tax on the contribution by the employer. Which of the following is correct? A) Since the assets were placed into a trust, the economic benefit doctrine will require inclusion in income for the current-year contributions made by the employer. B) Since Cindy Sue cannot receive the benefits until she has been with the employer for 10 years, the substantial risk of forfeiture doctrine will not require inclusion in income for the current-year contributions made by the employer. C) Since the assets were placed into a trust, the constructive receipt doctrine will require inclusion in income for the current year contributions made by the employer. D) Cindy Sue is subject to income tax in the current year because the plan is discriminatory.

B) Since Cindy Sue cannot receive the benefits until she has been with the employer for 10 years, the substantial risk of forfeiture doctrine will not require inclusion in income for the current-year contributions made by the employer. The economic benefit and constructive receipt doctrines will not cause inclusion because the assets are forfeitable if she does not stay with the company for the required length of service. Deferred compensation plans are by nature discriminatory. The contributions will be included if the employee has an economic benefit, no risk of forfeiture, or constructive receipt.

In terms of security of the Social Security system, which of the following statements is most accurate? A) Although Social Security is a federal program, it is administered by the individual states, and therefore its security is very dependent on the state. B) Social Security will face long-term financing shortfalls under currently scheduled benefits and financing. C) The security of Social Security system is stable because the benefits paid out in any one year are financed by the payroll taxes being paid in. D) It is estimated that the Social Security trust fund will be exhausted in the year 2023, and that will result in the elimination of Social Security benefits.

B) Social Security will face long-term financing shortfalls under currently scheduled benefits and financing. Social Security is a federally administered program that is financed through payroll taxes. The current concern is that with an aging population, the payroll taxes coming in may not fully sustain the benefits being paid out. This represents a shortfall, but not a cessation of benefits.

A SEP is not a qualified plan and is not subject to all of the qualified plan rules. However, it is subject to many of the same rules. Which of the following is a true statement? A) SEPs and qualified plans have different funding deadlines. B) The contribution limit for SEPs and qualified plans (defined contribution) C) SEPs and qualified plans have the same ERISA protection from creditors. D) SEPs and qualified plans have different nondiscriminatory and top-heavy rules.

B) The contribution limit for SEPs and qualified plans (defined contribution) SEPs and qualified plans can be funded as late as the due date of the return plus extensions. The maximum contribution for an individual to a SEP is $57,000 for 2020 ($285,000 maximum compensation × 25%, limited to $57,000). Thus statement B is correct. Qualified plans are protected under ERISA. IRAs and SEPs do not share this protection. Both types of plans have the same nondiscriminatory and top-heavy rules.

Part B of Medicare is considered supplemental insurance and provides additional coverage to participants. Which of the following is true regarding Part B coverage? A) The election to participate must be made at the time the insured is eligible for Part A Medicare and at no time after. B) The premiums for Part B are paid monthly through withholding from Social Security benefits. C) Once a participant elects Part B, he must maintain the coverage until death. D) Coverage under Part B does not include deductibles or coinsurance.

B) The premiums for Part B are paid monthly through withholding from Social Security benefits. Only Statement (B) is correct. Statement (A) is incorrect because participation can occur after the initial eligibility. Maintaining participation for life is not required, and Part B does have deductibles and/or coinsurance.

Which of the following expenditures will most likely increase during retirement? A) Clothing costs B) Travel C) FICA D) Savings

B) Travel Travel is the most likely expenditure to increase during retirement. Many other costs will likely be reduced after the retiree leaves the workforce, including a reduction in clothing expenses and the elimination of payroll taxes. An individual's savings may also be eliminated because a retirement plan requires the use of the accumulated savings during retirement.

Which of the following statements concerning cafeteria employee-benefit plans is correct? A) Cafeteria plans are the answer to the real world, where family benefit needs are homogeneous. B) Under a cafeteria plan, a family can write its own benefit plan by selecting among options available. C) The employer may offer educational assistance as a benefit under a cafeteria plan. D) Most cafeteria plans offer a pension plan or some type of deferred-compensation plan.

B) Under a cafeteria plan, a family can write its own benefit plan by selecting among options available. Statement (A) is not correct. Cafeteria plans are suited for groups in which family benefit needs are NOT homogeneous. Statement (C) is not a correct statement. Educational assistance is not an eligible benefit under a cafeteria plan. Statement (D) is not correct. 401(k) plans are the only type of retirement benefit plans that may be offered through a cafeteria plan.

All of the following are characteristics of pension plans EXCEPT: A) mandatory funding. B) in-service withdrawals for employees under the age of 59 ½ years. C) limited investment in life insurance. D) the limit of a 10 percent investment in the employer's securities.

B) in-service withdrawals for employees under the age of 59 ½ years. A pension plan requires mandatory funding, limits the investment of plan assets in life insurance, and limits the investment of plan assets in the employer's securities. A pension plan may not allow in-service withdrawals for employees under the age of 59½ years.

Carla would like to determine her financial needs during retirement. All of the following are costs she might eliminate in her retirement needs calculation EXCEPT: A) the $175 per month of parking expenses for parking at her place of employment. B) the $1,500 mortgage payment that is scheduled to end 2 years into retirement. C) the Medicare taxes she pays each year. D) the $2,000 per month she deposits into savings.

B) the $1,500 mortgage payment that is scheduled to end 2 years into retirement. Carla would not eliminate her mortgage since it will not be paid off at retirement. She would eliminate the parking expense, Medicare taxes, and savings expense since she would most likely no longer have these expenses during retirement.

Sew What, the best seamstress shop in town, sponsors a 401(k) plan. The plan provides a dollar-for-dollar match for employee contributions up to six percent and has immediate vesting for all contributions. For actual deferral percentage (ADP) purposes, the company has not made the top 20 percent election for the determination of who is highly compensated. The company has the following employee information: Lois: 93% Ownership $201,000 Compensation $14,000 Elective Deferral 6.97% Deferral Percentage Frank: 5% Ownership $150,000 Compensation $14,000 Elective Deferral 9.33% Deferral Percentage Karen: 2% Ownership $140,000 Compensation $12,600 Elective Deferral 9.00% Deferral Percentage Jeanette: ─ Ownership $40,000 Compensation $4,000 Elective Deferral 10.00% Deferral Percentage Joyce: −Ownership $30,000 Compensation ─ Elective Deferral 0.00% Deferral Percentage Ronnie: ─ Ownership $30,000 Compensation $1,800 Elective Deferral 6.00% Deferral Percentage Kali: − Ownership $30,000 Compensation − Elective Deferral 0.00% Deferral Percentage Which of the following statements is correct? A) Karen is not highly compensated. B) The plan passes the ADP test if Joyce and Kali were not eligible. C) Joyce and Kali are not considered when calculating the ADP test because they do not contribute. D) The top-heavy rules require Sew What to make a 3% contribution for Jeanette, Joyce, Ronnie, and Kali.

B)The plan passes the ADP test if Joyce and Kali were not eligible. Lois, Frank and Karen are highly compensated employees. Lois is the only one who meets the ownership test of greater than five percent. Lois, Frank, and Karen meet the earnings test for being highly compensated. The plan fails the ADP test. The NHC average ADP is 4% [(10% + 0% + 6% + 0%) / 4] while the HC ADP is 8.43% [(6.97% + 9.33% + 9.00%) / 3]. Thus using the test, 2% is added to the NHC ADP of 4%, which requires that the HC ADP be at or below 6%. Since the HC ADP is 8.43%, the plan fails the ADP test. If Joyce and Kali were not eligible, then the NHC ADP equals 8% and the plan meets the ADP test. Options (C) and (D) are not correct, since there is no indication that the plan is top heavy. In addition, top heavy applies to key employees, not highly compensated employees.

Which of the following are requirements for a qualified stock-bonus plan? I. Participants must have pass-through voting rights for stock held by the plan. II. Participants must have the right to demand employer securities as a distribution, even if the plan sponsor is a closely held corporation. A) I only B) II only C) Both I and II D) Neither I nor II

Both statements are true.

Larry, age 55, is employed by BB Trucking Company as a tire repair specialist. He earns $65,000 per year and received an allocation of $40,000 to his employer-provided profit-sharing plan for the year. If BB Trucking does not match employee deferrals, what is the maximum amount Larry can defer to his 401(k) plan for the 2020 plan year? A) $16,000 B) $19,000 C) $23,500 D) $25,000

C) $23,500 The maximum deferral to a 401(k) plan for a participant who is over 50 years old in 2020 is $26,000 ($19,500 plus catch-up of $6,500). The deferral is also included in the maximum defined-contribution limit of $57,000. Since Larry has received an allocation from the profit-sharing plan of $40,000, he is able to defer $17,000 ($57,000 − $40,000) plus the $6,500 catch-up deferral for participants who are 50 years old and older to maximize his defined-contribution plan limit for the year.

James, age 58, has compensation of $150,000 and wants to defer the maximum to his public 457(b) plan. The normal retirement age for his plan is age 60. How much can he defer in 2020 if he has an unused deferral amount of $60,000 from age 40 to age 49? A) $19,000 B) $25,000 C) $39,000 D) $44,000

C) $39,000 He can contribute $39,000 (2 × $19,500). He must be within three years of retirement and have unused deferral. Note that since he used the final 3-year catch-up, he cannot use the age-50-and-over catch-up.

Jordan contributed $5,000 each year to her Roth IRA for eleven years. At age 57, Jordan's IRA was worth $100,000, consisting of $55,000 in contributions, $25,000 in conversions from her 401(k) plan last year, and savings of $20,000. What are the tax consequences if Jordan takes a complete distribution of the Roth IRA at age 57, once she has retired, to travel around the world? A) Only $20,000 is subject to the 10 percent early withdrawal penalty. B) $20,000 is subject to income tax but no penalty. C) $45,000 is subject to the 10 percent early withdrawal penalty and $20,000 is subject to income tax. D) None of the distribution is taxable or subject to a penalty.

C) $45,000 is subject to the 10 percent early withdrawal penalty and $20,000 is subject to income tax. The distribution is not a qualified distribution, which means that Statement (D) is not correct. The distribution is not qualified, as it does not meet one of the distribution reasons, which is the attainment of age 59½. The $20,000 of earnings is subject to tax and penalty. The $25,000 of conversions is not subject to tax as it was already subject to income tax. However, because the conversion took place within the last five years, it is subject to the 10 percent penalty.

Contributing $1,500 to his retirement fund at the end of each year beginning at age 18 through age 50, with an average annual return of 12%, how much does Juan have in his retirement account at this time to use toward a possible early retirement? A) $346,766.42 B) $399,987.65 C) $457,271.58 D) $541,890.55

C) $457,271.58 N= 50 - 18 = 32 i = .12 PV= 0 Pmt= 1,500 FV= <457,271.5789>

Jim retired from PBS Industries, Inc. in 2015 at age 60 and began receiving a monthly defined-benefit pension of $7,520 as a single-life annuity. Unfortunately, the pension plan was badly underfunded and was taken over by PBGC in early 2020. Using the data below for the 2020 PBGC payout limits, which of the monthly payouts supplied in this table comes closest to what Jim will receive in the future? (Age at start of PBGC payout= Single-life Annuity) Age 60 = $3,778 Age 65 = $5,813 Age 70 = $9,649 A) $0 B) $3,778 C) $5,813 D) $7,520

C) $5,813 Badly underfunded pensions will not be allowed to continue paying out amounts in excess of the maximum for the year of a PBGC takeover. For a 65-year-old in 2020 the maximum payout for an underfunded plan is $5,812.50 monthly. Since Jim's monthly pension at age 65 of $7,520 is greater than the PBGC limit, it must be reduced to that age-65 limit. The age-60 PBGC limit doesn't apply because it is Jim's age at the time of takeover that matters.

Corey, age 54 and single, has compensation this year of $85,000. His employer does not sponsor a qualified plan, so Corey would like to contribute to a Roth IRA. What is Corey's maximum contribution for this year to the Roth IRA? A) $0 B) $6,000 C) $7,000 D) $21,250

C) $7,000 The maximum Roth IRA contribution is $6,000 plus $1,000 (2020) for those individuals aged 50 and over. Corey can make a $7,000 contribution to his Roth IRA. Corey's income does not exceed the phase-out limit for 2020.

A company's defined-benefit pension plan utilizes a funding formula that considers years of service and average compensation to determine the pension benefit payable to the plan participants. If Kim is a participant in this defined-benefit pension plan and she has 30 years of service with the company and always had annual compensation of $75,000, what is the maximum pension benefit that can be payable to Kim at her retirement? A)$19,000 B)$56,000 C)$75,000 D)$225,000

C) $75,000 The maximum amount payable from a defined benefit pension plan is the lesser of $230,000 (2020) or 100% of the average of the employee's 3 consecutive years of highest compensation. Because the average of Kim's compensation is $75,000, she would be limited to receiving a pension benefit at her retirement of $75,000.

Beth is the designated beneficiary of a Roth 401(k) plan that she inherited from her dad, Bob, who was 76 years old when he died in 2020. How should Beth calculate the required minimum distribution she will need to take for 2021? A) Non-recalculating single life expectancy method B) Non-recalculating uniform life expectancy method C) 10-year rule D) Distributions are not required for a Roth 401(k)

C) 10-year rule Based on the SECURE Act, Beth is a non-eligible designated beneficiary of the Roth 401(k) plan account. Since her father died in 2020, she must use the 10-year rule to determine her required minimum distributions. Statement (A) would have applied if Beth's dad had died in 2019. Statement (B) is incorrect because no such method exists. Statement (D) is incorrect because inherited Roth accounts are subject to RMDs.

What is the first year in which a single taxpayer, at age 48 in 2020, could receive a qualified distribution from a Roth IRA if he opened a Roth IRA for the first time on April 1, 2020 for the tax year 2019? A) 2022 B) 2023 C) 2024 D) 2031 (after he turns 59½)

C) 2024 A qualified distribution can only occur after a five-year period has occurred and is made on or after the date on which the owner attains age 59½, made to a beneficiary or the estate of the owner on or after the date of the owner's death, attributable to the owner's being disabled, or for a first-time home purchase. The five-year period starts at the beginning of the taxable year of the initial contribution to a Roth IRA. The five-year period ends on the last day of the individual's fifth consecutive taxable year beginning with the taxable year described in the preceding sentence. Thus 2024 is the first year in which a qualified distribution could occur.

Marcus has been employed by GCD Enterprises for 15 years, and currently earns $60,000 per year. Marcus saves $15,000 per year. He plans to pay off his home at retirement and live debt free. He currently spends $12,000 per year on his mortgage. What do you expect Marcus' wage replacement ratio to be, based on the above information? A) 28.41% B) 33.02% C) 47.35% D) 55.00%

C) 47.35% Calculate the wage replacement ratio: Salary: $60,000 100.00% Payroll Taxes: ($4,590) (7.65%) Savings: ($15,000) (25.00%) Mortgage Paid-off: ($12,000) (20.00%) $28,410 47.35%

In which of the following plans may an employer make tax-deductible contributions to a trust in the form of both principal and interest for a loan? A) A stock bonus plan B) An ESOP C) A leveraged ESOP D) An S corporation ESOP

C) A leveraged ESOP Only a leveraged ESOP will have a loan and thus have principal and interest payments.

Allison Quinn, age 43, is an employee of BizzCo Corporation and earns an annual salary of $95,000. Allison has recently been informed that the company is implementing a group term-life insurance plan and will pay the premium for an amount of coverage equal to her salary. Monthly cost for Allison from is 10 cents per $1,000 of death benefit exceeding minimum coverage. What are the tax implications of this benefit for Allison? A) Allison will not be taxed on the benefit as long as the plan is nondiscriminatory. B) Allison will pay tax on an additional $4.50 of income. C) Allison will pay tax on an additional $54.00 of income. D) Allison will pay tax on an additional $114.00 of income.

C) Allison will pay tax on an additional $54.00 of income. Employees are taxed on group term-life-insurance premiums paid for by the employer on death benefits over $50,000. The amount of additional income is calculated using the Table 1 cost of insurance, which shows the monthly cost per $1,000 of death benefit. Since Allison is 43 years old, the monthly cost from Table 1 is 10 cents per $1,000 of death benefit over $50,000. The employer is paying for an extra $45,000 of coverage: 45 × 0.10 × 12 = $54.

The Money-for-Life retirement income withdrawal strategy has the following advantage(s): 1) The overall portfolio from the combined tranches is similar to a moderately allocated portfolio. 2) This strategy helps the client focus on and differentiate current needs from future needs. A) I only B) II only C) Both I and II D) Neither I nor II

C) Both The Money-for-Life withdrawal strategy recognizes that retirees have both short- term and long- term needs. So, the first tranche might invest 25% of the portfolio in money market funds for the first 5 years, while the tranche for years 26--330 might have 12% of the portfolio invested in small-cap stocks in order to provide growth.

Which of following statements is (are) correct? I. An individually designed qualified plan will generally cost more for a company to establish than a prototype plan. II. A plan sponsor has the right to terminate a qualified retirement plan when the plan sponsor no longer has the funds available to continue to fund the plan at a reasonable level. A) I only B) II only C) Both I and II D) Neither I nor II

C) Both I and II An individually designed qualified plan will be drafted by attorneys to meet the specific needs and desires of the plan sponsor. A prototype plan is a standard or form plan that has already received a determination letter from the IRS. The plan sponsor simply chooses the appropriate elections for its plan. Because the individually designed qualified plan utilizes attorneys and the plan sponsor often requests a determination letter from the IRS, the costs are higher than those associated with a prototype plan. A plan sponsor can terminate a plan when the plan sponsor can no longer make contributions to the plan. In this case, all participants will become 100% vested in their accrued benefit.

Which of the following statements is (are) true? I. Participants of a stock-bonus plan sponsored by a C corporation must be given pass-through voting rights for the employer stock held by the plan. II. One disadvantage of an ESOP is that it may place an unnecessary cash-flow burden on the plan sponsor. A) I only B) II only C) Both I and II D) Neither I nor II

C) Both I and II Both statements are correct. Participants of a stock-bonus plan sponsored by a C corporation must be given pass-through voting rights for the employer stock held by the plan. An ESOP can create an unnecessary cash-flow burden for the plan sponsor in any of the following cases: (1) a participant terminates employment and requests a distribution of cash from the plan, (2) a participant attains the age of 55 and requests diversification within the plan, (3) the plan sponsor must pay for a periodic appraisal of the employer stock, and (4) a participant exercises his put/repurchase option.

Which of the following statements concerning profit-sharing plans is (are) correct? I. The employer can choose from a number of different allocation formulas. II. Profit-sharing plans are popular because contributions may be made on a discretionary basis. A) I only B) II only C) Both I and II D) Neither I nor II

C) Both I and II Both statements are true.

Which of the following statements regarding 457 plans is (are) true? I. An individual who defers $19,500 to his 403(b) plan during 2020 can also defer $19,500 to a 457 plan during 2020 (salary and plan permitting).II. A 457 plan allows an executive of a tax-exempt entity to defer compensation into an ERISA-protected trust.III. In the final three years before normal retirement age, a participant of a government sponsored 457 plan may be able to defer $39,000 (2020) for the plan year. A) I only B) II only C) I and III D) II and III

C) I and III Statement II is false. The funds deferred to a 457 plan established for a tax-exempt entity do not have ERISA protection. Statement I is true because the deferral limits for 403(b) plans and 457 plans are separate. Statement III is true as certain 457 plans allow the participants to defer twice the annual deferral limit in the last three years before the plan's normal retirement age.

Robbie is the owner of SS Automotive and he would like to establish a qualified pension plan. Robbie would like most of the plan's current contributions to be allocated to his account. He does not want to permit loans and he does not want SS Automotive to bear the investment risk of the plan's assets. Robbie is 32 and earns $700,000 per year. His employees are aged 25, 29, and 58 and they each earn $25,000 per year. Which of the following qualified pension plans would you recommend that Robbie establish? A) Target-benefit pension plan B) Cash-balance pension plan C) Money-purchase pension plan D) Defined-benefit pension plan using permitted disparity

C) Money-purchase pension plan Because Robbie does not want SS Automotive to bear the investment risk for the plan assets, the money-purchase pension plan or the target-benefit plan would be the available options to fulfill his requirements. The target-benefit plan would not fulfill Robbie's desires because as a percentage of compensation, older employees receive a greater contribution in a target-benefit plan and one of the employees is older than Robbie. In such a case, the older employee would receive a greater (as a percentage of compensation) contribution to the plan.

All of the following are reasons that an employer might favor a nonqualified plan over a qualified retirement plan EXCEPT: A) There is more design flexibility with a nonqualified plan. B) A nonqualified plan typically has lower administrative costs. C) Nonqualified plans typically allow the employer an immediate income tax deduction. D) Employers can generally exclude rank-and-file employees from a nonqualified plan.

C) Nonqualified plans typically allow the employer an immediate income tax deduction. Nonqualified plans do not allow the employer to take an income tax deduction until the employee recognizes the income. All of the other statements are correct.

Oliver is a 45-year old executive who earns $375,000 from his job at Acme Arrows (AA) and contributes the maximum amount to the 401(k) plan. He wants to make a contribution to a Roth IRA for the current year. Assume that Oliver has a traditional IRA with a balance of $10,000 that was funded entirely with pre-tax contributions. Which of the following is correct for 2020? A) Oliver cannot contribute to a Roth IRA because he is an active participant in his company's retirement plan. B) Oliver cannot execute the backdoor Roth strategy because he has a balance in a traditional IRA. C) Oliver could execute the backdoor Roth strategy but would have to recognize approximately $3,750 for income tax purposes if he contributes $6,000 to a traditional IRA. D) If Oliver executes a backdoor Roth strategy, he has the option to recharacterize the conversion if the value of the assets decreased prior to the filing of his return (including extensions).

C) Oliver could execute the backdoor Roth strategy but would have to recognize approximately $3,750 for income tax purposes if he contributes $6,000 to a traditional IRA. Oliver could execute a backdoor Roth but would have to recognize approximately $3,750 as income: 1 - ($6,000 / $16,000) × $6,000 = $3,750. Recharacterization of conversions is no longer permitted after 2017 as a result of the TCJA 2017.

Which of the following statements is true? A) Profit-sharing plans may not offer in-service withdrawals. B) Pension and profit-sharing plans are both subject to mandatory funding requirements. C) Profit-sharing plans allow annual employer contributions up to 25 percent of the employer's covered compensation. D) The legal promise of a profit-sharing plan is to pay a pension at retirement.

C) Profit-sharing plans allow annual employer contributions up to 25 percent of the employer's covered compensation. Statement (C) is the only true statement. Profit sharing plans allow annual contributions of up to 25 percent of covered compensation. Statement (A) is false because profit-sharing plans can allow in-service withdrawals. Statement (B) is false because while pension plans are subject to mandatory funding standards, profit-sharing plans are not. Statement (D) is false because the legal promise of a profit-sharing plan is the deferral of compensation, and the legal promise of a pension plan is to a pay a pension at retirement.

All of the following people would be considered a highly compensated employee for 2020 EXCEPT: A) Kim, a 1% owner whose salary last year was $150,000 B) Rita, a 6% owner whose salary was $42,000 last year C) Robin, an officer, who earned $105,000 last year and is the 29th highest paid employee of 96 employees D) Helen, who earned $132,000 last year and is in the top 20% of paid employees

C) Robin, an officer, who earned $105,000 last year and is the 29th highest paid employee of 96 employees Kim and Helen are highly compensated employees because their compensation was greater than $130,000. Rita is a highly compensated employee because she is a > 5% owner. Robin is not highly compensated, because she does not have compensation greater than $130,000.

Fred, aged 30, has a small but promising business with 5 current employees. He wishes to adopt an easily managed workplace plan that would allow him and his employees to defer modest amounts of their salary into the plan. Fred is not interested in maximizing his own deferrals, because he is plowing most of the earnings back into the business. He is concerned, however, about maintaining plan choice flexibility in case his business suddenly expands in size. Which of the following plans would Fred prefer to adopt? A) SEP IRA B) Profit sharing with 401(k) C) SIMPLE IRA D) Target-benefit pension plan

C) SIMPLE IRA A SIMPLE IRA would be ideal for Fred's business. Statement (A) is incorrect because SEP IRAs do not allow employee-salary deferrals. Statement (B) is incorrect because the additional complexity to maintain a qualified plan and higher deferral limits does not match Fred's requirements. Statement (D) is incorrect because target benefit pensions do not depend on employee salary deferrals.

All of the following are true regarding employer contributions to secular trusts for employee-participants of a nonqualified deferred-compensation agreement EXCEPT: A) Participants have security against an employer's unwillingness to pay at termination. B) Participants have security against an employer's bankruptcy. C) Secular trusts provide tax deferral for employees until distribution. D) Secular trusts provide employers with a current income tax deduction.

C) Secular trusts provide tax deferral for employees until distribution Secular trusts are similar to rabbi trusts except that participants do not have a substantial risk of forfeiture and thus do not provide the employee with tax deferral. Secular trusts provide the employer with a current income tax deduction for contributions. Secular trusts protect the participant from employer unwillingness to pay because they are funded. They also protect from bankruptcy because there is no risk of forfeiture.

All the following statements concerning Voluntary Employee Beneficiary Associations (VEBAs) are correct EXCEPT: A) The employer may deduct contributions for federal income tax purposes. B) The employer avoids taxation on the investment income earned on contributions. C) The employee may be provided retirement and deferred compensation benefits. D) Noncurrent employees may become members of a VEBA.

C) The employee may be provided retirement and deferred compensation benefits. Employees may not be provided retirement or deferred compensation benefits. Statements (A), (B), and (D) are correct statements. Note that the number of noncurrent employees that may become members of a VEBA cannot exceed 10% of the total membership.

Which of the following applies to key-person life insurance? A) The premiums are tax-deductible. B) The employee is the beneficiary. C) The employer pays the cost. D) The employee is the policy owner.

C) The employer pays the cost. The premiums are not tax-deductible. The employer is the owner, premium payer, and beneficiary. Therefore, statements (A), (B), and (D) are incorrect.

All of the following statements about ESOPs are correct EXCEPT: A) An ESOP is controlled through a trust. B) ESOPs provide corporate owners with a way to transfer ownership interests to their employees. C) The trust of an ESOP is prohibited from borrowing money from a bank to purchase the employer stock. D) Some conflicts of interests among employees, the corporation, and management of the corporation do not require the ESOP trustee to resign.

C) The trust of an ESOP is prohibited from borrowing money from a bank to purchase the employer stock. A key characteristic of the ESOP is that the trust may borrow money to purchase the employer stock.

Bob broke his leg skiing earlier this year. Bob collected $10,000 from his short-term disability plan paid for solely by his employer. How much does Bob have to include in his gross income? A) $0 B) $5,000 C) $8,500 D) $10,000

D) $10,000 Bob must include 100% of the disability premiums since his employer paid the entire premium.

Jennifer, age 54, earns $125,000 annually from ABC Incorporated. ABC sponsors a SIMPLE and matches all employee deferrals 100% up to a 3% contribution. Assuming Jennifer defers the maximum to her SIMPLE, what is the total contribution to the account in 2020 including both employee and employer contributions? A) $16,500 B) $16,750 C) $19,500 D) $20,250

D) $20,250 Jennifer can defer up to $16,500 ($13,500 + $3,000) for 2020 because she is over 50. ABC's match for Jennifer is 3% of her compensation, or $3,750 (3% × $125,000). The maximum contribution to Jennifer's SIMPLE is $20,250 ($13,500 + $3,000 + $3,750).

Paper Mill, Inc. provides a fringe benefit valued at $1,000 to its rank-and-file employees and a similar fringe benefit valued at $3,000 to its officers and highly compensated employees. The IRS has deemed this benefit to be discriminatory. Consequently, how much must each officer and highly compensated employee include in his gross income? A) $0 B) $1,000 C) $2,000 D) $3,000

D) $3,000 The plan is discriminatory to non-highly compensated employees; therefore, the entire value of the fringe benefit given to officers and highly compensated employees is includible in their gross income, not just the excess of what is available to the other employees.

Marisol was granted 100 NQSOs five years ago. At the time of the option grant, the value of the underlying stock was $100 and the exercise price was equal to $100. If Marisol exercises the options on August 22 of this year when the stock is valued at $145, what are the tax consequences (per share) to Marisol from exercising the options? A) $45 of W-2 income, $100 of short-term capital gain B) $100 of W-2 income, $45 of short-term capital gain C) $145 of W-2 income D) $45 of W-2 income

D) $45 of W-2 income At the exercise date of an NQSO, the individual will have to buy the stock at the exercise price and will have W-2 income for the stock value's appreciation amount in excess of the exercise price. In this case, Marisol will have $45 ($145 - $100) of W-2 income. There is no other gain or loss at exercise.

Billy's company sponsors a 401(k) profit-sharing plan with no employer match, but the company did make noncontributory employer contributions because the plan was top-heavy. Billy quit today after six years with the company and has come to you to determine how much of his retirement balance he can take with him. The plan uses the least generous graduated-vesting schedule available. What is Billy's vested account balance? Employer Contributions: $2,000 Employer Earnings: $600 Employee Contributions: $2,000 Employee Earnings: $600 A) $2,600 B) $4,160 C) $4,680 D) $5,200

D) $5,200 Billy is entitled to 100% of his contributions and the earnings on those contributions. The employer contributions, which were not matching contributions, will follow the least generous graduated-vesting schedule for a top-heavy plan. The least generous graduated vesting schedule is a 2-to-6-year graduated-vesting schedule for a 401(k) plan. At six years, Billy would be 100% vested in the employer contributions and the earnings on the employer contributions. Thus Billy's vested account balance is $5,200.

Jan, a 72-year-old widow, is a participant of the Ace Web Enterprise (AWE) 401(k) plan. For the current year, she deferred $6,000, or 10 percent, of her salary to the plan. She did not receive a profit-sharing contribution, a forfeiture, an employer match, or any other employer contribution. Jan would like to make a contribution to her Roth IRA. If Jan's AGI is $75,000 (all comprised of W-2 earnings, Social Security and portfolio income), what is the maximum contribution she can make for 2020? A) $0 B) $3,000 C) $6,000 D) $7,000

D) $7,000 Her income does not exceed the phase-out for contributing to a Roth IRA. Therefore, she can contribute the maximum amount, including the catch-up.

James is employed by a large corporation with 400 employees. The corporation provides its employees with a no-cost gym membership at the local public YMCA. The cost of the membership is $60/month, which is completely paid for by James' employer for all employees. How much, if any, must James include in his yearly gross income related to this fringe benefit? A) $0 B) $60 C) $600 D) $720

D) $720 James must include the full cost paid by his employer in his adjusted gross income. The exclusion for payment of health club facility dues is only provided when the facilities are on the employer's business premises and are solely for the use of the employees and their families. In this example, James' employer provides a membership at a public YMCA, so the fringe benefit is taxable.

Jason turned 72 in November of this year. He was a participant in his employer's profit-sharing plan. His profit-sharing plan had an account balance of $250,000 on December 31 of last year. According to the Uniform Lifetime Table, the factors for ages 70, 71, and 72 are 27.4, 26.5, and 25.6 respectively. What is Jason's required minimum distribution for this year? A) $7,300 B) $8,547 C) $8,812 D) $9,766

D) $9,766 Jason turned 70 ½ before January 1, 2020, and therefore is subject to the new RMD rules under the SECURE Act. He must take a required minimum distribution for this year because he turned 72. The distribution is calculated by taking the account balance at the end of last year divided by the factor for Jason's age at the end of this year. $250,000 / 25.6 = $9,766.

All of the following are permitted vesting schedules for a non-top-heavy profit-sharing plan EXCEPT: A) 2-to-6 year, graduated B) 3-year, cliff C) 1-to-4 year, graduated D) 3-to-7 year, cliff

D) 3-to-7 year, cliff As a result of the Pension Protection Act of 2006, a profit-sharing plan must vest at least as rapidly as a 3-year cliff, or 2-to-6-year graduated schedule without regard to the plan's top-heavy status. The profit-sharing plan can follow any vesting schedule that provides a more generous vesting schedule.

For part of the summer in the current year, Jordan worked full-time at a local accounting firm. Jordan is a 19-year-old accounting student who attends a state university and does not work during the rest of the year. Over the summer, Jordan earned a total of $9,500. How many quarters of coverage has Jordan attained for Social Security purposes based only on this year's earnings? A) None, since she did not work a full calendar quarter B) 1 quarter, the maximum she can earn in a 3-month period C) 2 quarters D) 4 quarters

D) 4 quarters In 2020, a worker earns one quarter of coverage for each $1,410 in earnings, up to a maximum of 4 quarters of coverage for the year. Since 1978, it is not necessary that the worker have earnings in each quarter - an annual income of 4 times the quarterly limit will attain 4 quarters of coverage for the year.

Which of the following statements is true? A) A cash-balance pension plan usually benefits older employees the most. B) A defined-benefit plan promises a contribution to a hypothetical account each year for a plan participant. C) Cash-balance pension-plan participants under the age of 59½ years may take a withdrawal from the plan during employment with the plan sponsor. D) A cash-balance pension plan does not have individual separate accounts for each participant.

D) A cash-balance pension plan does not have individual separate accounts for each participant. Statement (D) is true. The cash-balance pension plan consists of a commingled account that must be equal to the actuarial equivalent of the present value of the expected future benefits that will be paid from the cash-balance pension plan. Statement (A) is false because cash-balance pension plans usually benefit younger participants because of the guaranteed contribution rate and the guaranteed earnings rate. Statement (B) is false because a defined-benefit plan promises a defined benefit at the participant's retirement. If the funding requirements are met with plan earnings, a contribution is not required. Statement (C) is false because the participant of a cash-balance pension plan under the age of 59 ½ years cannot take in-service withdrawals.

Which of the following factors may affect a retirement plan? I. Career earnings II. Retirement life expectancy III. Mortality IV. Savings rate A) I and II B) II and III C) I, II, and IV D) All of the above

D) All of the above All of the options may affect a retirement plan either positively or negatively. Reduced work life expectancy may provide an insufficient savings period while an increased retirement life expectancy increases capital needs for retirement. A low savings rate may create an inability to meet capital requirements. Increased inflation rates will reduce purchasing power. The overall career earnings will affect the retirement need. As mortality increases, less is needed at retirement.

Which of the following situations would create an inclusion in an employee's gross income? A) Kay is the director and manager of Holiday Hotel. As a condition of her employment, Kay is required to live at the hotel. The value of this is $1,000 per month. B) Nate works at a fitness club. The club posts favorable comments about him on social media, and this increases Nate's private massage business by at least $500 in revenue each year. C) Brian is an airline pilot with We Don't Crash Airlines, Inc. and is allowed to fly as a passenger for free on the airline whenever an open seat is available. D) Eric, who is an active duty Marine, moved from Houston to New Orleans as was required by the Marines. His expenses for the move included $400 of truck rental costs, $100 of lodging and $200 of pre-move house hunting expenses. Eric's employer reimbursed him $600.

D) Eric, who is an active duty Marine, moved from Houston to New Orleans as was required by the Marines. His expenses for the move included $400 of truck rental costs, $100 of lodging and $200 of pre-move house hunting expenses. Eric's employer reimbursed him $600 Eric must include $100 of the reimbursement in his gross income as pre-move house hunting expenses that are not qualified moving expenses. To the extent that an individual is reimbursed for non-qualifying expenditures, the individual must include the reimbursement in his taxable income. In this case, the lodging of $100 and the truck rental of $400 are qualified expenses, so any reimbursement in excess of $500 ($400 + $100) is included in Eric's gross income. Since Eric's employer reimbursed him $600, $100 is taxable income to him. In statement (A), Kay is required to live at her employer's hotel as a condition of employment, thus the provision of the housing is not taxable. Statement (B) describes a benefit that is only theoretical in value. Statement (C) describes a "no-additional-cost benefit" that is not taxable to Brian. After 2017, moving expenses are not deductible except for active-duty members of the Armed Forces.

Which of the following qualified plan distributions are subject to a 10% early withdrawal penalty? I. Carolyn, aged 56, is currently employed by UBEIT Corporation and takes a $125,000 distribution from the UBEIT 401(k) plan. II. Brad, aged 60, takes a $1,000,000 distribution from his employer's profit-sharing plan. Ten days after receiving the $800,000 check (reduced for 20% withholding), Brad deposited the $800,000 into a new IRA account. III. Tara, aged 22, withdraws $2,000 of her contributions from her 401(k). A) I only B) III only C) II and III D) I and III

D) I and III Statement I is subjected to the 10% early withdrawal penalty, because Carolyn has not separated from service. Statement II will not be subjected to the 10% early withdrawal penalty because Brad is older than 59-1/2 years old. Statement III is subjected to the 10% penalty, because Tara does not qualify for any of the exclusions from the 10% penalty.

Organic, Inc., sponsors a qualified plan that requires employees to complete one year of service and be 21 years of age before entering the plan. The plan also excludes all commissioned sales people and all other allowable exclusions permitted under the code. Which of the following employees could be excluded? I. Sarah, age 32, who has been a secretary for the company for 11 months II. Andy, age 20, who works in accounting and has been with the company for 23 months III. Erin, a commissioned sales clerk, who works in the Atlanta office. Erin is 25 years old and has been with the company for 4 years. IV. George, age 29, who works in the factory. George has been with the company for 9 years and is covered under a collective bargaining agreement. A) I only B) I and III C) I, II, and IV D) I, II, III, and IV

D) I, II, III, and IV Each of these employees can be excluded from the plan. Sarah does not meet the service requirement. Andy does not meet the age requirement. Erin can be excluded because she is a commissioned salesperson. George is excluded because he is covered under a collective bargaining agreement.

ProLife, Inc., has a written adoption assistance program that pays adoption expenses including attorney and other fees, as well as other normal expenses. ProLife paid adoption expenses for the following employees during 2020. 1) Employee and Age 2) Employee AGI 3) Adoptee and Age 4) Amount Paid 5) Health of Adoptee 1) Joe (34) 2) $150,000 3) Cindy Lou (18) 4) $14,000 5) Excellent 1) James (34) 2) $260,000 3) Randi (6) 4) $14,000 5) Excellent 1) Donna (34) 2) $80,000 3) Brooke (3) 4) $14,000 5) Excellent 1) Connie (32) 2) $100,000 3)Silky (5) 4) $14,000 5) Excellent Which of the following have income inclusions resulting from the employer adoption plan? A) No one, as $14,300 is the maximum excludable limit for 2020. B) Joe C) James D) Joe and James

D) Joe and James Joe has adopted an 18-year-old whose health is excellent, so he must include the amount paid as income. An "eligible child" must be under the age of 18 or be physically or mentally incapable of caring for himself. James is over the income phaseout ($254,520 for 2020) and must include the $14,000 in income.

Which of the following statements is (are) correct regarding assets reverting back to the sponsor or a qualified plan? I. Under a merger, assets from a qualified plan can revert back to the plan sponsor without regard to the relationship between the value of the plan assets compared to the value of the obligations under the plan.II. Any reversion of plan assets will only be subject to a 20-percent penalty. A) I only B) II only C) Both I and II D) Neither I nor II

D) Neither I nor II Statement I is incorrect because the assets must exceed the plan liabilities. Statement II is incorrect because the penalty may be 20 percent or 50 percent.

One of the disadvantages of an ESOP is that the stock is an undiversified investment portfolio. Which of the following is correct? I. An employee, aged 65 or older, who has completed 10 years of participation in an ESOP may require that 25 percent of the account balance be diversified. II. An employee who receives corporate stock from an ESOP in a series of periodic distributions over 3 years may enjoy net unrealized appreciation treatment at the time of each distribution. A) I only B) II only C) Both I and II D) Neither I nor II

D) Neither I nor II The first statement is incorrect because the age is 55, not 65. The second statement is incorrect because net unrealized appreciation treatment is only available in the case when all the account assets are distributed within one tax year.

Which of the following statements regarding an age-based profit-sharing plan is correct? A) An age-based profit-sharing plan only provides a benefit to those plan participants whose age is within 10 years of the age of the owner of the plan sponsor. B) An age-based profit-sharing plan provides a greater benefit to those plan participants who are over 50 years old and whose earnings exceed the Social Security wage base. C) The majority of the profit-sharing allocation in an age-based profit-sharing plan is usually received by younger plan participants. D) Older plan participants receive greater benefits from an age-based profit-sharing plan.

D) Older plan participants receive greater benefits from an age-based profit-sharing plan. An age-based profit-sharing plan provides a greater benefit to older plan participants because the allocation of the plan contribution is based upon the age of the participants. Statements (A) and (B) are false statements without any merit. Statement (C) is incorrect because OLDER plan participants in an age-based profit-sharing plan usually receive the majority of the profit-sharing allocation.

All of the following are problems associated with Monte Carlo Analysis EXCEPT: A) Means and standard deviations for stock returns vary. B) It assumes normal distribution and serial independence for investment returns. C) Many Monte Carlo Analysis calculators ignore income tax consequences. D) The analysis only provides the most likely outcome.

D) The analysis only provides the most likely outcome. While a Monte Carlo Analysis has the problems listed in (A) through (C), the very advantage of the analysis is that is provides the most likely outcome and, typically, other possible outcomes.

Which of the following statements concerning prohibited transactions is true? A) The plan is free to lend its assets to company owners. B) The CPA auditor of the plan is never a disqualified person. C) The tax on prohibited transactions is 25% of the amount involved. D) The plan's investment advisor is a disqualified person.

D) The plan's investment advisor is a disqualified person. Any person providing services to a qualified plan is a disqualified person. Statement (A) is incorrect because a plan is prohibited from lending to a disqualified person. Statement (B) is incorrect because a plan service provider is always disqualified. Statement (C) is incorrect because the excise tax on prohibited transactions is 15% in the year of the transaction, plus an additional 100% if the transaction is not corrected within the taxable period.

To qualify for non-recognition of gain treatment, all of the following requirements apply EXCEPT: A) The ESOP must own at least 30% of the corporation's stock immediately after the sale. B) The corporation that establishes the ESOP must have no class of stock outstanding that is tradeable on an established securities market. C) The seller and 25% shareholders in the corporation are precluded from receiving allocations of stock acquired by the ESOP through the rollover. D) The stock sold to the ESOP must be common or convertible preferred stock and must have been owned by the seller for no more than three years prior to the sale.

D) The stock sold to the ESOP must be common or convertible preferred stock and must have been owned by the seller for no more than three years prior to the sale. The stock sold must have been owned by the seller for at least 3 years. All other statements are correct.

All of the following are common defined-benefit plan funding formulas EXCEPT: A) the flat amount formula. B) the flat percentage formula. C) the unit credit formula. D) the excludable amount formula.

D) the excludable amount formula. Answers (A), (B), and (C) are common defined-benefit-plan funding formulas. There is no such thing as an excludable amount formula.

Wallace and Associates is considering implementing a buy-sell agreement in which each partner purchases a life insurance policy on each of the other partners. Which one of the following statements is correct, given this information? A)The partners are entering into an entity redemption agreement. B)Upon the death of an owner, the life insurance proceeds will be used to buy out the decedent's share of the partnership. Those life insurance proceeds are taxable as ordinary income. C)The amount of insurance per policy will equal the value of the partnership. D)The partners are entering into a cross-purchase agreement.

D)The partners are entering into a cross-purchase agreement. Statement (A) is incorrect because the entity approach occurs when the business entity purchases life insurance policies on each owner. Statement (B) is incorrect because the life insurance proceeds are tax-exempt. Statement (C) is incorrect because each policy will be for the deceased partner's share.

Which of the following generally contributes to defined-benefit plans, profit-sharing plans, and money-purchase pension plans? A) Employees only B) Employer only C) Both employer and employees D) Employer, employees, and government

Employees contribute to 401(k) plans and thrift plans. The government does not contribute to plans, except when it is the employer.

In 2020, Chip, an accomplished professional race car driver, is to receive a signing bonus for agreeing to drive for Hot-Lap International, a racing team. Hot-Lap agrees to establish a NQDC agreement with Chip to defer the bonus beyond Chip's peak income producing years. Hot-Lap transfers the bonuses to an escrow agent, subject to the risk of forfeiture to team creditors in bankruptcy, who invests the funds in securities acting as a hedge against inflation. The bonus is deferred until 2021 and is then paid to Chip in years 2021-2030. When is the income deductible by the employer and includible by Chip? Option A: Employer Deduction: 2020 Employee Inclusion: 2020 Option B: Employer Deduction: 2021 Employee Inclusion: 2021 Option C: Employer Deduction: 2021-2030 Employee Inclusion: 2021 Option D: Employer Deduction: 2021-2030 Employee Inclusion: 2021-2030

Option D The income is only deductible when it is includible by Chip in 2021-2030.


संबंधित स्टडी सेट्स

Intermediate Accounting 2 Chapter 11 Part 2

View Set

Nomenclatura Tradicional de Óxidos Metálicos (III)

View Set

Econ 102 test 2 hw / clicker / pt questions

View Set

Chapter 3: Epigenetics and Disease

View Set

BLD 313 Final Exam Quiz Compilation

View Set

ati pharmacology practice assessment A

View Set

Chapter 14 - Planning Presentations

View Set

FINE 4110 Chp2.1-2.3 Asset Classes and Finance Firms

View Set